do eigenvectors correspond to direction of maximum scaling?












3












$begingroup$


Does the eigenvector correspond to a direction in which maximum scaling occurs by a given transformation matrix (A) acting upon this vector.
I quote from :
https://math.stackexchange.com/q/243553




No other vector when acted by this matrix will get stretched as much
as this eigenvector.




Is the above statement always true?... For example let
$$
A = left( begin{array}{ccc}
0.578385540014544 & 0.703045745965410 \
0.477513363789115 & 0.922698950982510 \
end{array} right)
$$



The largest eigenvalue is 1.35 (approx.)



Now, consider the vector (not eigenvector)
$$
v = left( begin{array}{ccc}
-0.538656963091298 \
-0.842525178326001 \
end{array} right)
$$



magnitude(v) = 1.0



magnitude(A*v) = 1.373



So this vector(v), which is not the eigenvector of A is scaled by a larger amount (x1.373), compared to the eigenvector which is scaled by x 1.35 (approx.)



Is this just an artifact of numerical precision ? I can easily create more examples of random square transformation matrices (A) where the eigenvector does not correspond to the direction of maximum scaling.










share|cite|improve this question











$endgroup$








  • 1




    $begingroup$
    The quote is a false statement. It is true for a symmetric matrix that the largest eigenvalue (in absolute value) equals the largest stretch factor. Your example shows that it's not necessarily true in general.
    $endgroup$
    – Greg Martin
    Sep 16 '15 at 6:59










  • $begingroup$
    Thank you for the answer Greg.
    $endgroup$
    – user1837245
    Sep 16 '15 at 19:58
















3












$begingroup$


Does the eigenvector correspond to a direction in which maximum scaling occurs by a given transformation matrix (A) acting upon this vector.
I quote from :
https://math.stackexchange.com/q/243553




No other vector when acted by this matrix will get stretched as much
as this eigenvector.




Is the above statement always true?... For example let
$$
A = left( begin{array}{ccc}
0.578385540014544 & 0.703045745965410 \
0.477513363789115 & 0.922698950982510 \
end{array} right)
$$



The largest eigenvalue is 1.35 (approx.)



Now, consider the vector (not eigenvector)
$$
v = left( begin{array}{ccc}
-0.538656963091298 \
-0.842525178326001 \
end{array} right)
$$



magnitude(v) = 1.0



magnitude(A*v) = 1.373



So this vector(v), which is not the eigenvector of A is scaled by a larger amount (x1.373), compared to the eigenvector which is scaled by x 1.35 (approx.)



Is this just an artifact of numerical precision ? I can easily create more examples of random square transformation matrices (A) where the eigenvector does not correspond to the direction of maximum scaling.










share|cite|improve this question











$endgroup$








  • 1




    $begingroup$
    The quote is a false statement. It is true for a symmetric matrix that the largest eigenvalue (in absolute value) equals the largest stretch factor. Your example shows that it's not necessarily true in general.
    $endgroup$
    – Greg Martin
    Sep 16 '15 at 6:59










  • $begingroup$
    Thank you for the answer Greg.
    $endgroup$
    – user1837245
    Sep 16 '15 at 19:58














3












3








3


3



$begingroup$


Does the eigenvector correspond to a direction in which maximum scaling occurs by a given transformation matrix (A) acting upon this vector.
I quote from :
https://math.stackexchange.com/q/243553




No other vector when acted by this matrix will get stretched as much
as this eigenvector.




Is the above statement always true?... For example let
$$
A = left( begin{array}{ccc}
0.578385540014544 & 0.703045745965410 \
0.477513363789115 & 0.922698950982510 \
end{array} right)
$$



The largest eigenvalue is 1.35 (approx.)



Now, consider the vector (not eigenvector)
$$
v = left( begin{array}{ccc}
-0.538656963091298 \
-0.842525178326001 \
end{array} right)
$$



magnitude(v) = 1.0



magnitude(A*v) = 1.373



So this vector(v), which is not the eigenvector of A is scaled by a larger amount (x1.373), compared to the eigenvector which is scaled by x 1.35 (approx.)



Is this just an artifact of numerical precision ? I can easily create more examples of random square transformation matrices (A) where the eigenvector does not correspond to the direction of maximum scaling.










share|cite|improve this question











$endgroup$




Does the eigenvector correspond to a direction in which maximum scaling occurs by a given transformation matrix (A) acting upon this vector.
I quote from :
https://math.stackexchange.com/q/243553




No other vector when acted by this matrix will get stretched as much
as this eigenvector.




Is the above statement always true?... For example let
$$
A = left( begin{array}{ccc}
0.578385540014544 & 0.703045745965410 \
0.477513363789115 & 0.922698950982510 \
end{array} right)
$$



The largest eigenvalue is 1.35 (approx.)



Now, consider the vector (not eigenvector)
$$
v = left( begin{array}{ccc}
-0.538656963091298 \
-0.842525178326001 \
end{array} right)
$$



magnitude(v) = 1.0



magnitude(A*v) = 1.373



So this vector(v), which is not the eigenvector of A is scaled by a larger amount (x1.373), compared to the eigenvector which is scaled by x 1.35 (approx.)



Is this just an artifact of numerical precision ? I can easily create more examples of random square transformation matrices (A) where the eigenvector does not correspond to the direction of maximum scaling.







eigenvalues-eigenvectors






share|cite|improve this question















share|cite|improve this question













share|cite|improve this question




share|cite|improve this question








edited Apr 13 '17 at 12:19









Community

1




1










asked Sep 16 '15 at 6:03









user1837245user1837245

162




162








  • 1




    $begingroup$
    The quote is a false statement. It is true for a symmetric matrix that the largest eigenvalue (in absolute value) equals the largest stretch factor. Your example shows that it's not necessarily true in general.
    $endgroup$
    – Greg Martin
    Sep 16 '15 at 6:59










  • $begingroup$
    Thank you for the answer Greg.
    $endgroup$
    – user1837245
    Sep 16 '15 at 19:58














  • 1




    $begingroup$
    The quote is a false statement. It is true for a symmetric matrix that the largest eigenvalue (in absolute value) equals the largest stretch factor. Your example shows that it's not necessarily true in general.
    $endgroup$
    – Greg Martin
    Sep 16 '15 at 6:59










  • $begingroup$
    Thank you for the answer Greg.
    $endgroup$
    – user1837245
    Sep 16 '15 at 19:58








1




1




$begingroup$
The quote is a false statement. It is true for a symmetric matrix that the largest eigenvalue (in absolute value) equals the largest stretch factor. Your example shows that it's not necessarily true in general.
$endgroup$
– Greg Martin
Sep 16 '15 at 6:59




$begingroup$
The quote is a false statement. It is true for a symmetric matrix that the largest eigenvalue (in absolute value) equals the largest stretch factor. Your example shows that it's not necessarily true in general.
$endgroup$
– Greg Martin
Sep 16 '15 at 6:59












$begingroup$
Thank you for the answer Greg.
$endgroup$
– user1837245
Sep 16 '15 at 19:58




$begingroup$
Thank you for the answer Greg.
$endgroup$
– user1837245
Sep 16 '15 at 19:58










1 Answer
1






active

oldest

votes


















1












$begingroup$

No - An eigenvector is the "input vector" of a matrix that "outputs" a vector in the same direction. If you really want an intuitive understanding of Eigenvalues and Eigenvectors, follow the link. (I don't have the reputation needed to post images.)



Linear Transformations & Eigenvectors Explained






share|cite|improve this answer











$endgroup$













    Your Answer





    StackExchange.ifUsing("editor", function () {
    return StackExchange.using("mathjaxEditing", function () {
    StackExchange.MarkdownEditor.creationCallbacks.add(function (editor, postfix) {
    StackExchange.mathjaxEditing.prepareWmdForMathJax(editor, postfix, [["$", "$"], ["\\(","\\)"]]);
    });
    });
    }, "mathjax-editing");

    StackExchange.ready(function() {
    var channelOptions = {
    tags: "".split(" "),
    id: "69"
    };
    initTagRenderer("".split(" "), "".split(" "), channelOptions);

    StackExchange.using("externalEditor", function() {
    // Have to fire editor after snippets, if snippets enabled
    if (StackExchange.settings.snippets.snippetsEnabled) {
    StackExchange.using("snippets", function() {
    createEditor();
    });
    }
    else {
    createEditor();
    }
    });

    function createEditor() {
    StackExchange.prepareEditor({
    heartbeatType: 'answer',
    autoActivateHeartbeat: false,
    convertImagesToLinks: true,
    noModals: true,
    showLowRepImageUploadWarning: true,
    reputationToPostImages: 10,
    bindNavPrevention: true,
    postfix: "",
    imageUploader: {
    brandingHtml: "Powered by u003ca class="icon-imgur-white" href="https://imgur.com/"u003eu003c/au003e",
    contentPolicyHtml: "User contributions licensed under u003ca href="https://creativecommons.org/licenses/by-sa/3.0/"u003ecc by-sa 3.0 with attribution requiredu003c/au003e u003ca href="https://stackoverflow.com/legal/content-policy"u003e(content policy)u003c/au003e",
    allowUrls: true
    },
    noCode: true, onDemand: true,
    discardSelector: ".discard-answer"
    ,immediatelyShowMarkdownHelp:true
    });


    }
    });














    draft saved

    draft discarded


















    StackExchange.ready(
    function () {
    StackExchange.openid.initPostLogin('.new-post-login', 'https%3a%2f%2fmath.stackexchange.com%2fquestions%2f1437569%2fdo-eigenvectors-correspond-to-direction-of-maximum-scaling%23new-answer', 'question_page');
    }
    );

    Post as a guest















    Required, but never shown

























    1 Answer
    1






    active

    oldest

    votes








    1 Answer
    1






    active

    oldest

    votes









    active

    oldest

    votes






    active

    oldest

    votes









    1












    $begingroup$

    No - An eigenvector is the "input vector" of a matrix that "outputs" a vector in the same direction. If you really want an intuitive understanding of Eigenvalues and Eigenvectors, follow the link. (I don't have the reputation needed to post images.)



    Linear Transformations & Eigenvectors Explained






    share|cite|improve this answer











    $endgroup$


















      1












      $begingroup$

      No - An eigenvector is the "input vector" of a matrix that "outputs" a vector in the same direction. If you really want an intuitive understanding of Eigenvalues and Eigenvectors, follow the link. (I don't have the reputation needed to post images.)



      Linear Transformations & Eigenvectors Explained






      share|cite|improve this answer











      $endgroup$
















        1












        1








        1





        $begingroup$

        No - An eigenvector is the "input vector" of a matrix that "outputs" a vector in the same direction. If you really want an intuitive understanding of Eigenvalues and Eigenvectors, follow the link. (I don't have the reputation needed to post images.)



        Linear Transformations & Eigenvectors Explained






        share|cite|improve this answer











        $endgroup$



        No - An eigenvector is the "input vector" of a matrix that "outputs" a vector in the same direction. If you really want an intuitive understanding of Eigenvalues and Eigenvectors, follow the link. (I don't have the reputation needed to post images.)



        Linear Transformations & Eigenvectors Explained







        share|cite|improve this answer














        share|cite|improve this answer



        share|cite|improve this answer








        edited Jan 5 at 14:59









        Bhavishya Desai

        32




        32










        answered Oct 12 '15 at 21:55









        user3527854user3527854

        111




        111






























            draft saved

            draft discarded




















































            Thanks for contributing an answer to Mathematics Stack Exchange!


            • Please be sure to answer the question. Provide details and share your research!

            But avoid



            • Asking for help, clarification, or responding to other answers.

            • Making statements based on opinion; back them up with references or personal experience.


            Use MathJax to format equations. MathJax reference.


            To learn more, see our tips on writing great answers.




            draft saved


            draft discarded














            StackExchange.ready(
            function () {
            StackExchange.openid.initPostLogin('.new-post-login', 'https%3a%2f%2fmath.stackexchange.com%2fquestions%2f1437569%2fdo-eigenvectors-correspond-to-direction-of-maximum-scaling%23new-answer', 'question_page');
            }
            );

            Post as a guest















            Required, but never shown





















































            Required, but never shown














            Required, but never shown












            Required, but never shown







            Required, but never shown

































            Required, but never shown














            Required, but never shown












            Required, but never shown







            Required, but never shown







            Popular posts from this blog

            Can a sorcerer learn a 5th-level spell early by creating spell slots using the Font of Magic feature?

            Does disintegrating a polymorphed enemy still kill it after the 2018 errata?

            A Topological Invariant for $pi_3(U(n))$